Simplify with Trigonometric Identities

Click For Summary
The discussion focuses on simplifying trigonometric expressions related to a physics problem involving a rod with two masses. The user is struggling to apply trigonometric identities, particularly sin²x + cos²x = 1, due to the presence of different angles and terms like dø²*sin²Θ. Suggestions include factoring out common terms, such as cos²θ, to simplify the expression further. Participants emphasize the importance of identifying common factors in the terms to aid in simplification. The conversation highlights the collaborative effort to clarify the application of trigonometric identities in complex problems.
KleZMeR
Messages
125
Reaction score
0

Homework Statement




I'm trying to simplify some trigonometric expressions, I'm attaching my work here. This comes from a famous physics problem i.e. the rod with two masses spinning on a circle. I've tried many times but I just can't get it. Any help on which identities to use would really help me.



Homework Equations



I'm attaching the simplified version, the 'answer'


The Attempt at a Solution



I'm attaching my work.
 

Attachments

  • photo (4).JPG
    photo (4).JPG
    31.1 KB · Views: 454
  • Screen Shot 2014-09-06 at 2.12.59 AM.png
    Screen Shot 2014-09-06 at 2.12.59 AM.png
    1.5 KB · Views: 467
Physics news on Phys.org
Apply a couple of times sin2 x + cos2 x = 1
and your last line simplifies to the given answer
 
NascentOxygen said:
Apply a couple of times sin2 x + cos2 x = 1
and your last line simplifies to the given answer

Yes, I looked at that identity, but there are 2 different angles in this problem, and also one of my last two terms is dø2*sin2Θ

Any direction on how to first apply the identity would also help?
 
Take cos2 θ outside some brackets because it's a common factor.

And you'll see again, there's another common factor in two other terms to treat similarly.
 
Question: A clock's minute hand has length 4 and its hour hand has length 3. What is the distance between the tips at the moment when it is increasing most rapidly?(Putnam Exam Question) Answer: Making assumption that both the hands moves at constant angular velocities, the answer is ## \sqrt{7} .## But don't you think this assumption is somewhat doubtful and wrong?

Similar threads

  • · Replies 7 ·
Replies
7
Views
2K
  • · Replies 18 ·
Replies
18
Views
3K
  • · Replies 13 ·
Replies
13
Views
2K
  • · Replies 2 ·
Replies
2
Views
1K
  • · Replies 30 ·
2
Replies
30
Views
3K
  • · Replies 1 ·
Replies
1
Views
1K
  • · Replies 4 ·
Replies
4
Views
1K
  • · Replies 1 ·
Replies
1
Views
1K
  • · Replies 1 ·
Replies
1
Views
1K
  • · Replies 14 ·
Replies
14
Views
2K